LSAT and Law School Admissions Forum

Get expert LSAT preparation and law school admissions advice from PowerScore Test Preparation.

 Brook Miscoski
PowerScore Staff
  • PowerScore Staff
  • Posts: 418
  • Joined: Sep 13, 2018
|
#58892
Howard,

I believe that you are asking how to tell that the last sentence is the conclusion and that the claim that "willingness to pay is not proportional to need" is not the conclusion.

I think that the easiest way, not necessarily the only way, is to compare the two sentences and ask yourself which one takes a step farther. The last sentence makes the additional claim that price increases will allocate goods to people with more money, not to people with more need. That's why you eliminate (B), because the last sentence is the main conclusion.

Another reason is that the last sentence is where the stimulus reaches its rejection of the economists' position, so that last sentence is the conclusion. You are set up by the overall structure and tone to expect the conclusion to directly oppose the economists.

An indicator is also "as a result," which is a good start but not where you finish because it can be used for an intermediate conclusion. Remember, you are correct to be looking for those indicators, but you still need to confirm. If you have nothing else to go on, it's a strong chance that those indicators show where the main conclusion is.
 LSAT2018
  • Posts: 242
  • Joined: Jan 10, 2018
|
#59119
James Finch wrote:Hi M,

The difference between answer choices (A) and (E) lies in the way the argument is structured. The consumer advocate is arguing against the economists' claim that price gouging is efficient by denying the premise upon which it is based, that willingness to pay corresponds with need. If answer (A) were correct, the consumer advocate woudl be agreeing with the economists' claim, but for a different reason. (E) is correct because it mirrors both the denial of the economists' premise ("denies a claim") and rejection of the conclusion ("reasoning it rejects").

Hope this helps!
So if answer (A) were correct, the consumer advocate would be saying pricing gouging is efficient because _____ (an alternative explanation). But the advocate doesn't seem to say that pricing gouging is efficient. Instead, he corrects the economists' assumption that price gouging allocates goods to people whose willingness to pay more shows that they need those goods.
 Adam Tyson
PowerScore Staff
  • PowerScore Staff
  • Posts: 5191
  • Joined: Apr 14, 2011
|
#59125
Correct! Disagreeing with an explanation means coming up with alternatives that do a better job of reaching the same conclusion, or supporting the same circumstances. In this stimulus, the author came to an opposing conclusion, rather than just finding a better way to reach the same one.
 Pragmatism
  • Posts: 68
  • Joined: Jan 11, 2018
|
#62575
I incorrectly chose B as an answer choice assuming it to be the main conclusion. After reviewing the thread, I am uncertain as to how a “claim” is being constituted as a premise, when my belief was that it meant a conclusion.
James Finch wrote:Hi M,

The difference between answer choices (A) and (E) lies in the way the argument is structured. The consumer advocate is arguing against the economists' claim that price gouging is efficient by denying the premise upon which it is based, that willingness to pay corresponds with need. If answer (A) were correct, the consumer advocate woudl be agreeing with the economists' claim, but for a different reason. (E) is correct because it mirrors both the denial of the economists' premise ("denies a claim") and rejection of the conclusion ("reasoning it rejects").

Hope this helps!
Can someone please clarify that point.

So, the way I am reading answer choice E is as follows:

—denies a claim (refusing to accept the economist’s conclusion that price gouging is efficient because no alternative seller can provide the needed goods) that the argument takes to be assumed (I am assuming it means, consumers’ willingness to pay is not proportional to [their] need, which is illustrated in the real world sentence) in the reasoning that it rejects (the premise stating: consumers willingness to pay more shows that they really need those goods, whereas the consumer advocates falsely equivocates “really need those goods” with “those with the most need”).
 Robert Carroll
PowerScore Staff
  • PowerScore Staff
  • Posts: 1787
  • Joined: Dec 06, 2013
|
#62587
Pragmatism,

A "claim" is anything that the author (or anyone else, for that matter) states is the case. It may thus be a premise, a conclusion, or just completely extraneous information. If the author claims that a certain thing is true, then infers another statement from that initial claim, the claim is a premise. If the author claims that a certain things is true because another thing is true, then the claim is a conclusion. "Claim" does not universally refer to the same part of an argument in every case. A claim is really just something the author puts forward as a true statement.

When answer choice (E) refers to the "claim" that the consumer advocate denies, that claim is a purported assumption of the contrary argument. The conclusion of the contrary argument is that price gouging is efficient. The reasoning behind that contrary argument relies on an assumption about a true connection between willingness to pay and need. The "claim" denied by the consumer advocate is that assumption. Because the contrary argument (at least according to the consumer advocate) relies on that assumption, the advocate thinks he/she can prove that argument wrong by knocking down the assumption. So the advocate denies that willingness to pay is proportional to need (the assumption) in order to show that price gouging is not efficient. Because the economists were arguing that price gouging IS efficient, this constitutes a rejection of their conclusion.

If you realize that a claim can be a premise, I think your misunderstanding will disappear.

Robert Carroll
 Pragmatism
  • Posts: 68
  • Joined: Jan 11, 2018
|
#62607
Thank you :-D
 ronibass
  • Posts: 15
  • Joined: Jun 18, 2019
|
#71775
I had answer choice E as a contender but I rejected it because of the words "takes to be assumed". My question is what exactly is the assumption that is being made in the stimulus? Thanks!
 Adam Tyson
PowerScore Staff
  • PowerScore Staff
  • Posts: 5191
  • Joined: Apr 14, 2011
|
#71843
In the argument that the author is rejecting, ronibass, the Economists must be assuming that willingness to pay is proportional to need. That is, the more you need something, the more you will pay for it. Our author explicitly rejects that assumption and thereby rejects the entire argument made by the Economists. What a painfully worded answer choice! It's designed to make our heads hurt, for sure.

Get the most out of your LSAT Prep Plus subscription.

Analyze and track your performance with our Testing and Analytics Package.